===== Teoremas de ponto fixo ===== Provavelmente você vai querer saber os resultados da [[lista:CauchyCompletude|lista de sequências de Cauchy e completude]]. Seja $(X, \tau)$ um espaço topológico. Dizemos que $x \in X$ é um {{entry>ponto/fixo; ponto fixo}} para $f: X \rightarrow X$ se $f(x) = x$. Sejam $(X, d)$ e $(Y, d')$ espaços métricos. Dizemos que $f: X \rightarrow Y$ é uma {{entry>contração}} se existe $k \in [0, 1[$ tal que, para todo $x, y$ distintos, $d'(f(x), f(y)) \leq k d(x, y)$. **~~#~~** Mostre que toda contração é uma função contínua. **~~#~~** Seja $(X, d)$ espaço métrico e seja $f: X \rightarrow X$ uma contração. Mostre que, se $f$ admite um ponto fixo, ele é único. **~~#~~** Seja $(X, d)$ espaço métrico completo e seja $f: X \rightarrow X$ uma contração. Seja $x_0 \in X$. Para cada $n \in \mathbb N_{>0}$, defina $x_{n + 1} = f(x_n)$. **~~#.#~~** Seja $n \in \mathbb N$. Mostre que $d(x_n, x_{n + 1}) \leq k^n d(x_0, x_1)$. **~~#.#~~** Sejam $n, p \in \mathbb N$. Mostre que $d(x_n, x_{n + p}) \leq \frac{k^n}{1 - k} d(x_0, x_1)$. **~~#.#~~** Conclua que $(x_n)_{n \in \mathbb N}$ é uma sequência de Cauchy e, portanto, convergente para algum $x \in X$. Note que tal $x$ é um ponto fixo. **~~#.#~~** Note que você mostrou o {{entry>Teorema/ponto fixo de Banach; Teorema do ponto fixo de Banach}}: Dado $(X, d)$ métrico completo e $f: X \rightarrow X$ contração, então $f$ admite um único ponto fixo $x$. Além disso, dado $x_0 \in X$, se definimos $x_{n + 1} = f(x_n)$ para todo $n \in \mathbb N$, temos que $x_n \rightarrow x$. Vamos ver que a hipótese $k < 1$ para a contração é essencial para o teorema do ponto fixo de Banach. Também veremos que nem tudo está perdido se retirarmos isso, mas daí você provavelmente vai querer saber os resultados da [[lista:compactos|lista de compactos]]. **~~#~~** Considere $X = [1, +\infty[$ com a métrica usual. Considere $f: X \rightarrow X$ dada por $f(x) = x + \frac{1}{x}$. **~~#.#~~** Note que $X$ é completo. **~~#.#~~** Note que $f$ não admite ponto fixo. **~~#.#~~** Note que $f$ é estritamente crescente. **~~#.#~~** Mostre que, dado $x, y \in X$ distintos, temos que $d(f(x), f(y)) < d(x, y)$. **~~#~~** Seja $(X, d)$ compacto. Seja $f: X \rightarrow X$ tal que $d(f(x), f(y)) < d(x, y)$ se $x \neq y$. **~~#.#~~** Note que $f$ é contínua. **~~#.#~~** Note que, se $f$ tem ponto fixo, então ele é único. **~~#.#~~** Note que $g: X \rightarrow \mathbb R$ dada por $g(x) = d(x, f(x))$ é contínua. **~~#.#~~** Mostre que $f$ tem um ponto fixo.